Démonstrations élégantes

Un problème, une question, un nouveau théorème ?

Messages : 0

Inscription : 28 août 2019 22:46

Profil de l'utilisateur : Élève de lycée

Re: Démonstrations élégantes

Message par Calli » 04 juil. 2021 20:43

T'as écrit $q$ au lieu de $q^{n-1}$ vers le début de ta démonstration.

Messages : 172

Inscription : 11 mars 2021 18:24

Profil de l'utilisateur : Élève de lycée

Re: Démonstrations élégantes

Message par Contrexemple » 08 juil. 2021 18:59

Calculer $$A=\int_0^1 \dfrac{\sqrt{x}} {\sqrt{x} +\sqrt{1-x}} dx$$
$ $
SPOILER:
Par un changement de variable $ t=1-x$, on obtient

$A=\int_0^1 \dfrac{\sqrt{1-x}}{\sqrt{x}+\sqrt{1-x}}dx$

Donc $2A=\int_0^1 \dfrac{\sqrt{1-x}+\sqrt{x}}{\sqrt{x}+\sqrt{1-x}} dx=1$

En fait on a même pour toute fonction continue par morceaux $f$

$\int_0^1 \dfrac{f(x)} {f(x) +f(1-x)} dx = 1/2$

Messages : 172

Inscription : 11 mars 2021 18:24

Profil de l'utilisateur : Élève de lycée

Re: Démonstrations élégantes

Message par Contrexemple » 10 juil. 2021 13:20

Déterminer une CNS sur les coeffs d'un polynôme de deg 3 pour qu'il soit bijectif sur $\mathbb F_p$, $p >5 $ nombre premier.
SPOILER:
L'intêret de cette question, c'est de reconnaître facilement les polynômes bijectifs, de deg 3 et ainsi savoir si on peut déterminer, facilement, le nombre de point de la courbe elliptique associée.

On prend P de la forme : $P(x)=x^3+ax+b$
On peut toujours ramené n'importe quelle polynôme de deg 3 à cette forme (sans perdre la bijectivité, si bijectivité il y a), par un changement de variable affine $x=y+c$, $c$ à bien choisir pour virer le terme quadratique.

On suppose $a \neq 0$ (c'est le cas le plus délicat à traiter)

cas : $-a$ est un carré dans $\mathbb F_p$, $e$ la racine carré non nul (car $a$ non nul)
Alors $P(e)=P(0)=b$ donc $P$ n'est pas bijectif.

Cas : $-a$ n'est pas un carré
$P(x)=P(y)$ et $x \neq y$
ssi $x^3-y^3+a(x-y)=0$ et $x-y\neq 0$ ssi $x^2-xy+y^2+a=0$ et $x-y\neq 0$
ssi $(\dfrac{x+y}{2})^2+3(\dfrac{x-y}{2})^2=-a $ et $x-y\neq 0$
ssi $u^2+3v^2=-a$ et $v\neq 0$

les applications $f: x \rightarrow -x^2$ et $g: x\rightarrow 3x^2+a$ sont telles que $card(f(\mathbb F_p))=card(g(\mathbb F_p))=(p+1)/2$ dans $\mathbb F_p$ de card $p$.
Donc $g(\mathbb F_p) \cap f(\mathbb F_p) \neq \emptyset$
$u,v \in \mathbb F_p$ tel que $-u^2=3v^2+a $ comme $-a$ n'est pas un carré on n'a pas $v=0$.

Donc $P$ n'est pas bijective.

Messages : 172

Inscription : 11 mars 2021 18:24

Profil de l'utilisateur : Élève de lycée

Re: Démonstrations élégantes

Message par Contrexemple » 11 juil. 2021 16:40

Calculer : $$A=\sum \limits_{n \geq 1} \dfrac{1}{n(n+2)(n+3)(n+5)(n+7)(n+11)}=\sum\limits_{n\geq 1} u_n$$
SPOILER:
Pour faire ce calcul, on va se servir d'une généralisation des sommes telescopiques.
En utilisant la décomposition en pôle simple on obtient
On a $u_n=\dfrac{a_0}{n}+\dfrac{a_2}{n+2}+\dfrac{a_3}{n+3}+\dfrac{a_5}{n+5}+\dfrac{a_7}{n+7}+\dfrac{a_{11}}{n+11}$ (*)

Avec $\sum \limits_{i=0}^{11} a_i=0$ où les $a_i$ n'intervenant pas dans (*) pris égaux à 0.

Donc $A_N=\sum \limits_{n=1}^N \sum \limits_{i=0}^{11} \dfrac{a_i}{n+i}=\sum\limits_{i=0}^{11} a_i\sum \limits_{n=1}^N \dfrac{1}{n+i}$

$A_N=\sum \limits_{i=0}^{11} a_i(\sum\limits_{n=13}^{N-1} \dfrac{1}{n}+\sum
\limits_{k=1+i}^{12} \dfrac{1}{k}+\sum \limits_{k=N}^{N+i} \dfrac{1}{k} )$

$$A_N=\sum\limits_{i=0}^{11} a_i(\sum\limits_{k=i+1}^{12} \dfrac{1}{k}+\sum \limits_{k=N}^{N+i} \dfrac{1}{k} ) $$

Messages : 172

Inscription : 11 mars 2021 18:24

Profil de l'utilisateur : Élève de lycée

Re: Démonstrations élégantes

Message par Contrexemple » 14 juil. 2021 15:05

Soit $\mathbb K$ un corps fini d'ordre $q$, $E=\mathbb K^n$ ev dont $(e_1,..,e_n)$ est une base.

On note $H_i=vect\{e_k; k=1...n \text{ et } k\neq i\}$. Déterminer $card(\bigcup\limits_{i=1}^n H_i)=card(W)$.
SPOILER:

$V=E-W$ le complémentaire de $W$ est l'ensemble $V=\{x=\sum \limits_{k=1}^n a_k e_k ; \forall k=1..n, a_k\in \mathbb K^*\}$

Donc $V=(q-1)^n$ d'où $W=q^n-(q-1)^n$

Je trouve cette démonstration élégante car, si on ne pense pas à passer par le complémentaire (formule d'inclusion exclusion du cardinal*) le calcul du cardinal devient beaucoup plus délicat.

* : nous verrons,, une manière élégante d'utiliser ce principe

Messages : 172

Inscription : 11 mars 2021 18:24

Profil de l'utilisateur : Élève de lycée

Re: Démonstrations élégantes

Message par Contrexemple » 14 juil. 2021 15:28

Soit $P(x, y)\in \mathbb N[x, y] $

On dispose d'un oracle qui nous donne la valeur de $P(x, y) $ pour $ x, y$ rationnel de notre choix.

Combien de nombres de questions minimums à l oracle, sont ils nécessaires pour déterminer entièrement $P$ ?
SPOILER:

En fait 2 questions sont nécessaires et suffises : $m=P(2,2)$ est plus grand que les coeffs de $P$ et que son degré.

et $P(m+1,(m+1)^{m+1})$ en regardant cette entier en base $(m+1)$ on obtient toutes les infos

Remarque : on peut étendre la méthode à un nombre de variables quelconques

Messages : 172

Inscription : 11 mars 2021 18:24

Profil de l'utilisateur : Élève de lycée

Re: Démonstrations élégantes

Message par Contrexemple » 16 juil. 2021 16:54

Calculer $$C=\sum\limits_{A\subset \{1,..,n\}, A\neq \emptyset } (-1)^{|A|+1}\min\{a: a\in A\} $$
SPOILER:

C'est l'application directe du principe d'inclusion exclusion à : $$card(\bigcup \limits_{i=1}^n H_i) $$
$ $ avec $H_1=\{1\},H_2=\{1,2\},...,H_n=\{1,...,n\}$

Donc $C=n$

Mais le meilleur reste à venir,, avec un usage, encore plus astucieux de ce résultat.

Exo sympa :

Calculer $$C=\sum\limits_{A\subset \{1,..,n\}, A\neq \emptyset } (-1)^{|A|+1}\max\{a: a\in A\}$$


Si vous séchez trop longtemps, demander la correction (y compris par Mp)...

Messages : 0

Inscription : 28 août 2019 22:46

Profil de l'utilisateur : Élève de lycée

Re: Démonstrations élégantes

Message par Calli » 16 juil. 2021 23:13

Une preuve exactement similaire à viewtopic.php?f=3&t=18640&start=6900#p1025145 peut être donnée à l'exercice précédent (n prenant le rôle de 1).

Messages : 0

Inscription : 28 août 2019 22:46

Profil de l'utilisateur : Élève de lycée

Re: Démonstrations élégantes

Message par Calli » 16 juil. 2021 23:15

C'est quoi la preuve de l'exo avec le max à laquelle tu pensais contrexemple ?

Messages : 172

Inscription : 11 mars 2021 18:24

Profil de l'utilisateur : Élève de lycée

Re: Démonstrations élégantes

Message par Contrexemple » 17 juil. 2021 00:05

Pour la plus part des énigmes que je propose, il n'arrive pas souvent que l'on re-tombe sur la solution que j'avais en tête.

C'est ce qui me fait croire pour des problèmes réputés difficiles (par exemple le théorème de Thompson-Feit) il existe sûrement des solutions simples auxquelles personne encore n'a pensées.

SPOILER:

$$n=\sum\limits_{A\subset \{1,..,n\}, A\neq \emptyset } (-1)^{|A|+1}\min\{a: a\in A\}=\sum\limits_{A\subset \{1,..,n\}, A\neq \emptyset } (-1)^{|A|+1}\min\{n+1-a: a\in A\}=\sum\limits_{A\subset \{1,..,n\}, A\neq \emptyset } (-1)^{|A|+1}(n+1-\max\{a: a\in A\})$$

Répondre